Identically distributed vs P(X > Y) = P(Y > X)












3














I've two related propositions which seem correct intuitively, but I struggle to prove them properly.



Question 1



Prove or disprove: If $X$ and $Y$ are independent and identically distributed, then $mathbb{P} (Y > X) = mathbb{P} (X > Y) = 1/2$



Due to independence, the joint PDF of $X$ and $Y$ is the product of their marginal PDF:



$$ begin{align}
mathbb{P} (Y > X) &= int_{-infty}^infty int_x^infty p(x) , p(y) , dy , dx \
mathbb{P} (X > Y) &= int_{-infty}^infty int_y^infty p(x) , p(y) , dx , dy
= int_{-infty}^infty int_x^infty p(y) , p(x) , dy , dx
end{align} $$



The last step is based on the fact that the integral won't change if we simply rename the integration parameters $x$ and $y$ consistently. So we have shown that $mathbb{P} (Y > X) = mathbb{P} (X > Y)$



Side note: Even if $X$ and $Y$ are dependent, this result still holds so long as their joint PDF is exchangeable i.e. $p(x, y) = p(y, x)$





Let $u = y - x$ so that



$$ mathbb{P} (Y > X) = int_{-infty}^infty int_0^infty p(x) , p(u + x) , du , dx $$



I thought of applying Fubini's theorem but it doesn't help to show that it's equal to 1/2, so maybe it's not 1/2?



Alternatively, consider that



$$ mathbb{P} (Y > X) + mathbb{P} (X > Y) + mathbb{P} (Y = X) = 1 $$



If we assume that $mathbb{P} (Y = X) = 0$ then we can conclude that $mathbb{P} (Y > X) = 1/2$. But is this assumption justified?



Question 2



Prove or disprove: If $X$ and $Y$ are independent and $mathbb{P} (Y > X) = mathbb{P} (X > Y)$, then they are identically distributed.



If this is true, then is it still true if $X$ and $Y$ are dependent?










share|cite|improve this question
























  • One interesting technical point is that if the probability of $X=c$ is $0$ for every value of $c,$ that's not enough to entail that probabilities are given by integrating a density function. The standard counterexample is the Cantor distribution. But more to the point$,ldotsqquad$
    – Michael Hardy
    6 hours ago










  • $ldots,$is that I probably wouldn't solve this problem by considering such integrals anyway.
    – Michael Hardy
    6 hours ago
















3














I've two related propositions which seem correct intuitively, but I struggle to prove them properly.



Question 1



Prove or disprove: If $X$ and $Y$ are independent and identically distributed, then $mathbb{P} (Y > X) = mathbb{P} (X > Y) = 1/2$



Due to independence, the joint PDF of $X$ and $Y$ is the product of their marginal PDF:



$$ begin{align}
mathbb{P} (Y > X) &= int_{-infty}^infty int_x^infty p(x) , p(y) , dy , dx \
mathbb{P} (X > Y) &= int_{-infty}^infty int_y^infty p(x) , p(y) , dx , dy
= int_{-infty}^infty int_x^infty p(y) , p(x) , dy , dx
end{align} $$



The last step is based on the fact that the integral won't change if we simply rename the integration parameters $x$ and $y$ consistently. So we have shown that $mathbb{P} (Y > X) = mathbb{P} (X > Y)$



Side note: Even if $X$ and $Y$ are dependent, this result still holds so long as their joint PDF is exchangeable i.e. $p(x, y) = p(y, x)$





Let $u = y - x$ so that



$$ mathbb{P} (Y > X) = int_{-infty}^infty int_0^infty p(x) , p(u + x) , du , dx $$



I thought of applying Fubini's theorem but it doesn't help to show that it's equal to 1/2, so maybe it's not 1/2?



Alternatively, consider that



$$ mathbb{P} (Y > X) + mathbb{P} (X > Y) + mathbb{P} (Y = X) = 1 $$



If we assume that $mathbb{P} (Y = X) = 0$ then we can conclude that $mathbb{P} (Y > X) = 1/2$. But is this assumption justified?



Question 2



Prove or disprove: If $X$ and $Y$ are independent and $mathbb{P} (Y > X) = mathbb{P} (X > Y)$, then they are identically distributed.



If this is true, then is it still true if $X$ and $Y$ are dependent?










share|cite|improve this question
























  • One interesting technical point is that if the probability of $X=c$ is $0$ for every value of $c,$ that's not enough to entail that probabilities are given by integrating a density function. The standard counterexample is the Cantor distribution. But more to the point$,ldotsqquad$
    – Michael Hardy
    6 hours ago










  • $ldots,$is that I probably wouldn't solve this problem by considering such integrals anyway.
    – Michael Hardy
    6 hours ago














3












3








3


1





I've two related propositions which seem correct intuitively, but I struggle to prove them properly.



Question 1



Prove or disprove: If $X$ and $Y$ are independent and identically distributed, then $mathbb{P} (Y > X) = mathbb{P} (X > Y) = 1/2$



Due to independence, the joint PDF of $X$ and $Y$ is the product of their marginal PDF:



$$ begin{align}
mathbb{P} (Y > X) &= int_{-infty}^infty int_x^infty p(x) , p(y) , dy , dx \
mathbb{P} (X > Y) &= int_{-infty}^infty int_y^infty p(x) , p(y) , dx , dy
= int_{-infty}^infty int_x^infty p(y) , p(x) , dy , dx
end{align} $$



The last step is based on the fact that the integral won't change if we simply rename the integration parameters $x$ and $y$ consistently. So we have shown that $mathbb{P} (Y > X) = mathbb{P} (X > Y)$



Side note: Even if $X$ and $Y$ are dependent, this result still holds so long as their joint PDF is exchangeable i.e. $p(x, y) = p(y, x)$





Let $u = y - x$ so that



$$ mathbb{P} (Y > X) = int_{-infty}^infty int_0^infty p(x) , p(u + x) , du , dx $$



I thought of applying Fubini's theorem but it doesn't help to show that it's equal to 1/2, so maybe it's not 1/2?



Alternatively, consider that



$$ mathbb{P} (Y > X) + mathbb{P} (X > Y) + mathbb{P} (Y = X) = 1 $$



If we assume that $mathbb{P} (Y = X) = 0$ then we can conclude that $mathbb{P} (Y > X) = 1/2$. But is this assumption justified?



Question 2



Prove or disprove: If $X$ and $Y$ are independent and $mathbb{P} (Y > X) = mathbb{P} (X > Y)$, then they are identically distributed.



If this is true, then is it still true if $X$ and $Y$ are dependent?










share|cite|improve this question















I've two related propositions which seem correct intuitively, but I struggle to prove them properly.



Question 1



Prove or disprove: If $X$ and $Y$ are independent and identically distributed, then $mathbb{P} (Y > X) = mathbb{P} (X > Y) = 1/2$



Due to independence, the joint PDF of $X$ and $Y$ is the product of their marginal PDF:



$$ begin{align}
mathbb{P} (Y > X) &= int_{-infty}^infty int_x^infty p(x) , p(y) , dy , dx \
mathbb{P} (X > Y) &= int_{-infty}^infty int_y^infty p(x) , p(y) , dx , dy
= int_{-infty}^infty int_x^infty p(y) , p(x) , dy , dx
end{align} $$



The last step is based on the fact that the integral won't change if we simply rename the integration parameters $x$ and $y$ consistently. So we have shown that $mathbb{P} (Y > X) = mathbb{P} (X > Y)$



Side note: Even if $X$ and $Y$ are dependent, this result still holds so long as their joint PDF is exchangeable i.e. $p(x, y) = p(y, x)$





Let $u = y - x$ so that



$$ mathbb{P} (Y > X) = int_{-infty}^infty int_0^infty p(x) , p(u + x) , du , dx $$



I thought of applying Fubini's theorem but it doesn't help to show that it's equal to 1/2, so maybe it's not 1/2?



Alternatively, consider that



$$ mathbb{P} (Y > X) + mathbb{P} (X > Y) + mathbb{P} (Y = X) = 1 $$



If we assume that $mathbb{P} (Y = X) = 0$ then we can conclude that $mathbb{P} (Y > X) = 1/2$. But is this assumption justified?



Question 2



Prove or disprove: If $X$ and $Y$ are independent and $mathbb{P} (Y > X) = mathbb{P} (X > Y)$, then they are identically distributed.



If this is true, then is it still true if $X$ and $Y$ are dependent?







joint-distribution iid symmetry






share|cite|improve this question















share|cite|improve this question













share|cite|improve this question




share|cite|improve this question








edited 1 hour ago

























asked 6 hours ago









farmer

566




566












  • One interesting technical point is that if the probability of $X=c$ is $0$ for every value of $c,$ that's not enough to entail that probabilities are given by integrating a density function. The standard counterexample is the Cantor distribution. But more to the point$,ldotsqquad$
    – Michael Hardy
    6 hours ago










  • $ldots,$is that I probably wouldn't solve this problem by considering such integrals anyway.
    – Michael Hardy
    6 hours ago


















  • One interesting technical point is that if the probability of $X=c$ is $0$ for every value of $c,$ that's not enough to entail that probabilities are given by integrating a density function. The standard counterexample is the Cantor distribution. But more to the point$,ldotsqquad$
    – Michael Hardy
    6 hours ago










  • $ldots,$is that I probably wouldn't solve this problem by considering such integrals anyway.
    – Michael Hardy
    6 hours ago
















One interesting technical point is that if the probability of $X=c$ is $0$ for every value of $c,$ that's not enough to entail that probabilities are given by integrating a density function. The standard counterexample is the Cantor distribution. But more to the point$,ldotsqquad$
– Michael Hardy
6 hours ago




One interesting technical point is that if the probability of $X=c$ is $0$ for every value of $c,$ that's not enough to entail that probabilities are given by integrating a density function. The standard counterexample is the Cantor distribution. But more to the point$,ldotsqquad$
– Michael Hardy
6 hours ago












$ldots,$is that I probably wouldn't solve this problem by considering such integrals anyway.
– Michael Hardy
6 hours ago




$ldots,$is that I probably wouldn't solve this problem by considering such integrals anyway.
– Michael Hardy
6 hours ago










2 Answers
2






active

oldest

votes


















2














Question 1: I assume you mean to ask whether$$mathbb{P}(X<Y)=mathbb{P}(Y<X)tag{1}$$because, the way the question is written$$mathbb{P}(X<Y)=mathbb{P}(Y>X)tag{2}$$is always true since $X<Y$ and $Y>X$ are the same events. Considering (1), it is sufficient that $X$ and $Y$ are exchangeable, that is, that $(X,Y)$ and $(Y,X)$ have the same joint distribution for (1) to hold. And obviously
$$mathbb{P}(X<Y)=mathbb{P}(Y<X)=1/2$$since they sum up to one.



Question 2: Take a bivariate normal vector $(X,Y)$ with mean $(mu,mu)$. Then $X-Y$ and $Y-X$ are identically distributed, no matter what the correlation between $X$ and $Y$, and no matter what the variances of $X$ and $Y$ are, and therefore (1) holds. The conjecture is thus false.






share|cite|improve this answer





















  • Thanks, I've fixed the typo in my question.
    – farmer
    1 hour ago










  • For Q1, you seem to assume that P(Y = X) = 0. Is this justified? For Q2, I can't see why that's a counter-example. To falsify it, we need to find X, Y such that (1) holds but they are not identically distributed.
    – farmer
    58 mins ago



















1














I will show that the distribution of the pair $(X,Y)$ is the same as the distribution of the pair $(Y,X).$



That two random variables $X,Y$ are independent means that for every pair of measurable sets $A,B$ the events $[Xin A], [Yin B]$ are independent. In particular for any two numbers $x,y$ the events $[Xle x], [Yle y]$ are independent, so $F_{X,Y}(x,y) = F_X(x)cdot F_Y(y).$



And the distribution of the pair $(X,Y)$ is completely determined by the joint c.d.f.



Since it is given that $F_X=F_Y,$ we can write $F_{X,Y}(x,y) = F_X(x)cdot F_X(y).$



This is symmetric as a function of $x$ and $y,$ i.e. it remains the same if $x$ and $y$ are interchanged.



But interchanging $x$ and $y$ in $F_{X,Y}(x,y)$ is the same as interchanging $X$ and $Y,$ since
$$
F_{X,Y}(x,y) = Pr(Xle x & Yle y).
$$



Therefore (the main point):



The distribution of the pair $(X,Y)$ is the same as the distribution of the pair $(Y,X).$






share|cite|improve this answer























    Your Answer





    StackExchange.ifUsing("editor", function () {
    return StackExchange.using("mathjaxEditing", function () {
    StackExchange.MarkdownEditor.creationCallbacks.add(function (editor, postfix) {
    StackExchange.mathjaxEditing.prepareWmdForMathJax(editor, postfix, [["$", "$"], ["\\(","\\)"]]);
    });
    });
    }, "mathjax-editing");

    StackExchange.ready(function() {
    var channelOptions = {
    tags: "".split(" "),
    id: "65"
    };
    initTagRenderer("".split(" "), "".split(" "), channelOptions);

    StackExchange.using("externalEditor", function() {
    // Have to fire editor after snippets, if snippets enabled
    if (StackExchange.settings.snippets.snippetsEnabled) {
    StackExchange.using("snippets", function() {
    createEditor();
    });
    }
    else {
    createEditor();
    }
    });

    function createEditor() {
    StackExchange.prepareEditor({
    heartbeatType: 'answer',
    autoActivateHeartbeat: false,
    convertImagesToLinks: false,
    noModals: true,
    showLowRepImageUploadWarning: true,
    reputationToPostImages: null,
    bindNavPrevention: true,
    postfix: "",
    imageUploader: {
    brandingHtml: "Powered by u003ca class="icon-imgur-white" href="https://imgur.com/"u003eu003c/au003e",
    contentPolicyHtml: "User contributions licensed under u003ca href="https://creativecommons.org/licenses/by-sa/3.0/"u003ecc by-sa 3.0 with attribution requiredu003c/au003e u003ca href="https://stackoverflow.com/legal/content-policy"u003e(content policy)u003c/au003e",
    allowUrls: true
    },
    onDemand: true,
    discardSelector: ".discard-answer"
    ,immediatelyShowMarkdownHelp:true
    });


    }
    });














    draft saved

    draft discarded


















    StackExchange.ready(
    function () {
    StackExchange.openid.initPostLogin('.new-post-login', 'https%3a%2f%2fstats.stackexchange.com%2fquestions%2f385885%2fidentically-distributed-vs-px-y-py-x%23new-answer', 'question_page');
    }
    );

    Post as a guest















    Required, but never shown

























    2 Answers
    2






    active

    oldest

    votes








    2 Answers
    2






    active

    oldest

    votes









    active

    oldest

    votes






    active

    oldest

    votes









    2














    Question 1: I assume you mean to ask whether$$mathbb{P}(X<Y)=mathbb{P}(Y<X)tag{1}$$because, the way the question is written$$mathbb{P}(X<Y)=mathbb{P}(Y>X)tag{2}$$is always true since $X<Y$ and $Y>X$ are the same events. Considering (1), it is sufficient that $X$ and $Y$ are exchangeable, that is, that $(X,Y)$ and $(Y,X)$ have the same joint distribution for (1) to hold. And obviously
    $$mathbb{P}(X<Y)=mathbb{P}(Y<X)=1/2$$since they sum up to one.



    Question 2: Take a bivariate normal vector $(X,Y)$ with mean $(mu,mu)$. Then $X-Y$ and $Y-X$ are identically distributed, no matter what the correlation between $X$ and $Y$, and no matter what the variances of $X$ and $Y$ are, and therefore (1) holds. The conjecture is thus false.






    share|cite|improve this answer





















    • Thanks, I've fixed the typo in my question.
      – farmer
      1 hour ago










    • For Q1, you seem to assume that P(Y = X) = 0. Is this justified? For Q2, I can't see why that's a counter-example. To falsify it, we need to find X, Y such that (1) holds but they are not identically distributed.
      – farmer
      58 mins ago
















    2














    Question 1: I assume you mean to ask whether$$mathbb{P}(X<Y)=mathbb{P}(Y<X)tag{1}$$because, the way the question is written$$mathbb{P}(X<Y)=mathbb{P}(Y>X)tag{2}$$is always true since $X<Y$ and $Y>X$ are the same events. Considering (1), it is sufficient that $X$ and $Y$ are exchangeable, that is, that $(X,Y)$ and $(Y,X)$ have the same joint distribution for (1) to hold. And obviously
    $$mathbb{P}(X<Y)=mathbb{P}(Y<X)=1/2$$since they sum up to one.



    Question 2: Take a bivariate normal vector $(X,Y)$ with mean $(mu,mu)$. Then $X-Y$ and $Y-X$ are identically distributed, no matter what the correlation between $X$ and $Y$, and no matter what the variances of $X$ and $Y$ are, and therefore (1) holds. The conjecture is thus false.






    share|cite|improve this answer





















    • Thanks, I've fixed the typo in my question.
      – farmer
      1 hour ago










    • For Q1, you seem to assume that P(Y = X) = 0. Is this justified? For Q2, I can't see why that's a counter-example. To falsify it, we need to find X, Y such that (1) holds but they are not identically distributed.
      – farmer
      58 mins ago














    2












    2








    2






    Question 1: I assume you mean to ask whether$$mathbb{P}(X<Y)=mathbb{P}(Y<X)tag{1}$$because, the way the question is written$$mathbb{P}(X<Y)=mathbb{P}(Y>X)tag{2}$$is always true since $X<Y$ and $Y>X$ are the same events. Considering (1), it is sufficient that $X$ and $Y$ are exchangeable, that is, that $(X,Y)$ and $(Y,X)$ have the same joint distribution for (1) to hold. And obviously
    $$mathbb{P}(X<Y)=mathbb{P}(Y<X)=1/2$$since they sum up to one.



    Question 2: Take a bivariate normal vector $(X,Y)$ with mean $(mu,mu)$. Then $X-Y$ and $Y-X$ are identically distributed, no matter what the correlation between $X$ and $Y$, and no matter what the variances of $X$ and $Y$ are, and therefore (1) holds. The conjecture is thus false.






    share|cite|improve this answer












    Question 1: I assume you mean to ask whether$$mathbb{P}(X<Y)=mathbb{P}(Y<X)tag{1}$$because, the way the question is written$$mathbb{P}(X<Y)=mathbb{P}(Y>X)tag{2}$$is always true since $X<Y$ and $Y>X$ are the same events. Considering (1), it is sufficient that $X$ and $Y$ are exchangeable, that is, that $(X,Y)$ and $(Y,X)$ have the same joint distribution for (1) to hold. And obviously
    $$mathbb{P}(X<Y)=mathbb{P}(Y<X)=1/2$$since they sum up to one.



    Question 2: Take a bivariate normal vector $(X,Y)$ with mean $(mu,mu)$. Then $X-Y$ and $Y-X$ are identically distributed, no matter what the correlation between $X$ and $Y$, and no matter what the variances of $X$ and $Y$ are, and therefore (1) holds. The conjecture is thus false.







    share|cite|improve this answer












    share|cite|improve this answer



    share|cite|improve this answer










    answered 5 hours ago









    Xi'an

    53.9k690348




    53.9k690348












    • Thanks, I've fixed the typo in my question.
      – farmer
      1 hour ago










    • For Q1, you seem to assume that P(Y = X) = 0. Is this justified? For Q2, I can't see why that's a counter-example. To falsify it, we need to find X, Y such that (1) holds but they are not identically distributed.
      – farmer
      58 mins ago


















    • Thanks, I've fixed the typo in my question.
      – farmer
      1 hour ago










    • For Q1, you seem to assume that P(Y = X) = 0. Is this justified? For Q2, I can't see why that's a counter-example. To falsify it, we need to find X, Y such that (1) holds but they are not identically distributed.
      – farmer
      58 mins ago
















    Thanks, I've fixed the typo in my question.
    – farmer
    1 hour ago




    Thanks, I've fixed the typo in my question.
    – farmer
    1 hour ago












    For Q1, you seem to assume that P(Y = X) = 0. Is this justified? For Q2, I can't see why that's a counter-example. To falsify it, we need to find X, Y such that (1) holds but they are not identically distributed.
    – farmer
    58 mins ago




    For Q1, you seem to assume that P(Y = X) = 0. Is this justified? For Q2, I can't see why that's a counter-example. To falsify it, we need to find X, Y such that (1) holds but they are not identically distributed.
    – farmer
    58 mins ago













    1














    I will show that the distribution of the pair $(X,Y)$ is the same as the distribution of the pair $(Y,X).$



    That two random variables $X,Y$ are independent means that for every pair of measurable sets $A,B$ the events $[Xin A], [Yin B]$ are independent. In particular for any two numbers $x,y$ the events $[Xle x], [Yle y]$ are independent, so $F_{X,Y}(x,y) = F_X(x)cdot F_Y(y).$



    And the distribution of the pair $(X,Y)$ is completely determined by the joint c.d.f.



    Since it is given that $F_X=F_Y,$ we can write $F_{X,Y}(x,y) = F_X(x)cdot F_X(y).$



    This is symmetric as a function of $x$ and $y,$ i.e. it remains the same if $x$ and $y$ are interchanged.



    But interchanging $x$ and $y$ in $F_{X,Y}(x,y)$ is the same as interchanging $X$ and $Y,$ since
    $$
    F_{X,Y}(x,y) = Pr(Xle x & Yle y).
    $$



    Therefore (the main point):



    The distribution of the pair $(X,Y)$ is the same as the distribution of the pair $(Y,X).$






    share|cite|improve this answer




























      1














      I will show that the distribution of the pair $(X,Y)$ is the same as the distribution of the pair $(Y,X).$



      That two random variables $X,Y$ are independent means that for every pair of measurable sets $A,B$ the events $[Xin A], [Yin B]$ are independent. In particular for any two numbers $x,y$ the events $[Xle x], [Yle y]$ are independent, so $F_{X,Y}(x,y) = F_X(x)cdot F_Y(y).$



      And the distribution of the pair $(X,Y)$ is completely determined by the joint c.d.f.



      Since it is given that $F_X=F_Y,$ we can write $F_{X,Y}(x,y) = F_X(x)cdot F_X(y).$



      This is symmetric as a function of $x$ and $y,$ i.e. it remains the same if $x$ and $y$ are interchanged.



      But interchanging $x$ and $y$ in $F_{X,Y}(x,y)$ is the same as interchanging $X$ and $Y,$ since
      $$
      F_{X,Y}(x,y) = Pr(Xle x & Yle y).
      $$



      Therefore (the main point):



      The distribution of the pair $(X,Y)$ is the same as the distribution of the pair $(Y,X).$






      share|cite|improve this answer


























        1












        1








        1






        I will show that the distribution of the pair $(X,Y)$ is the same as the distribution of the pair $(Y,X).$



        That two random variables $X,Y$ are independent means that for every pair of measurable sets $A,B$ the events $[Xin A], [Yin B]$ are independent. In particular for any two numbers $x,y$ the events $[Xle x], [Yle y]$ are independent, so $F_{X,Y}(x,y) = F_X(x)cdot F_Y(y).$



        And the distribution of the pair $(X,Y)$ is completely determined by the joint c.d.f.



        Since it is given that $F_X=F_Y,$ we can write $F_{X,Y}(x,y) = F_X(x)cdot F_X(y).$



        This is symmetric as a function of $x$ and $y,$ i.e. it remains the same if $x$ and $y$ are interchanged.



        But interchanging $x$ and $y$ in $F_{X,Y}(x,y)$ is the same as interchanging $X$ and $Y,$ since
        $$
        F_{X,Y}(x,y) = Pr(Xle x & Yle y).
        $$



        Therefore (the main point):



        The distribution of the pair $(X,Y)$ is the same as the distribution of the pair $(Y,X).$






        share|cite|improve this answer














        I will show that the distribution of the pair $(X,Y)$ is the same as the distribution of the pair $(Y,X).$



        That two random variables $X,Y$ are independent means that for every pair of measurable sets $A,B$ the events $[Xin A], [Yin B]$ are independent. In particular for any two numbers $x,y$ the events $[Xle x], [Yle y]$ are independent, so $F_{X,Y}(x,y) = F_X(x)cdot F_Y(y).$



        And the distribution of the pair $(X,Y)$ is completely determined by the joint c.d.f.



        Since it is given that $F_X=F_Y,$ we can write $F_{X,Y}(x,y) = F_X(x)cdot F_X(y).$



        This is symmetric as a function of $x$ and $y,$ i.e. it remains the same if $x$ and $y$ are interchanged.



        But interchanging $x$ and $y$ in $F_{X,Y}(x,y)$ is the same as interchanging $X$ and $Y,$ since
        $$
        F_{X,Y}(x,y) = Pr(Xle x & Yle y).
        $$



        Therefore (the main point):



        The distribution of the pair $(X,Y)$ is the same as the distribution of the pair $(Y,X).$







        share|cite|improve this answer














        share|cite|improve this answer



        share|cite|improve this answer








        edited 5 hours ago

























        answered 5 hours ago









        Michael Hardy

        3,6451430




        3,6451430






























            draft saved

            draft discarded




















































            Thanks for contributing an answer to Cross Validated!


            • Please be sure to answer the question. Provide details and share your research!

            But avoid



            • Asking for help, clarification, or responding to other answers.

            • Making statements based on opinion; back them up with references or personal experience.


            Use MathJax to format equations. MathJax reference.


            To learn more, see our tips on writing great answers.





            Some of your past answers have not been well-received, and you're in danger of being blocked from answering.


            Please pay close attention to the following guidance:


            • Please be sure to answer the question. Provide details and share your research!

            But avoid



            • Asking for help, clarification, or responding to other answers.

            • Making statements based on opinion; back them up with references or personal experience.


            To learn more, see our tips on writing great answers.




            draft saved


            draft discarded














            StackExchange.ready(
            function () {
            StackExchange.openid.initPostLogin('.new-post-login', 'https%3a%2f%2fstats.stackexchange.com%2fquestions%2f385885%2fidentically-distributed-vs-px-y-py-x%23new-answer', 'question_page');
            }
            );

            Post as a guest















            Required, but never shown





















































            Required, but never shown














            Required, but never shown












            Required, but never shown







            Required, but never shown

































            Required, but never shown














            Required, but never shown












            Required, but never shown







            Required, but never shown







            Popular posts from this blog

            Contact image not getting when fetch all contact list from iPhone by CNContact

            count number of partitions of a set with n elements into k subsets

            A CLEAN and SIMPLE way to add appendices to Table of Contents and bookmarks